Difference between revisions of "2021 Fall AMC 12B Problems/Problem 3"

(Solution 1)
(Solution 1)
Line 15: Line 15:
 
<math>(M-5)+2=(L+3)</math>
 
<math>(M-5)+2=(L+3)</math>
  
Plug in <math>M=L+N</math> to solve the two equations respectively to get <math>N=10</math> or <math>N=6</math>. Hence the answer is <math>60 \Right arrow\boxed{(\textbf{C})  }.</math>
+
Plug in <math>M=L+N</math> to solve the two equations respectively to get <math>N=10</math> or <math>N=6</math>. Hence the answer is <math>60 \Right arrow \boxed{(\textbf{C})  }.</math>
  
 
~Wilhelm Z
 
~Wilhelm Z

Revision as of 02:53, 24 November 2021

Problem

At noon on a certain day, Minneapolis is $N$ degrees warmer than St. Louis. At $4{:}00$ the temperature in Minneapolis has fallen by $5$ degrees while the temperature in St. Louis has risen by $3$ degrees, at which time the temperatures in the two cities differ by $2$ degrees. What is the product of all possible values of $N?$

$(\textbf{A})\: 10\qquad(\textbf{B}) \: 30\qquad(\textbf{C}) \: 60\qquad(\textbf{D}) \: 100\qquad(\textbf{E}) \: 120$

Solution 1

Let the temperature of Minneapolis be $M$, and that of St. Louis be $L$. We have $M=L+N$.

At $4{:}00$, either

$(M-5)=2+(L+3)$

or

$(M-5)+2=(L+3)$

Plug in $M=L+N$ to solve the two equations respectively to get $N=10$ or $N=6$. Hence the answer is $60 \Right arrow \boxed{(\textbf{C}) }.$ (Error compiling LaTeX. Unknown error_msg)

~Wilhelm Z

2021 Fall AMC 12B (ProblemsAnswer KeyResources)
Preceded by
Problem 2
Followed by
Problem 4
1 2 3 4 5 6 7 8 9 10 11 12 13 14 15 16 17 18 19 20 21 22 23 24 25
All AMC 12 Problems and Solutions

The problems on this page are copyrighted by the Mathematical Association of America's American Mathematics Competitions. AMC logo.png